How Do You Find a Basis for the Null Space of Matrix A?

  • Thread starter Thread starter Derill03
  • Start date Start date
  • Tags Tags
    Basis Matrix
Click For Summary
To find a basis for the null space of matrix A, one must solve the equation AX = 0, where X is the vector of variables [x1, x2, x3, x4]^t. The solution space is identified as a two-dimensional subspace of R4, indicating it spans a plane through the origin. A basis consists of vectors that can be combined to form any solution vector within this space. To determine the basis, solve the given equations and express some variables in terms of others, selecting simple values for the free variables to find specific solutions. Understanding these concepts is crucial for effectively approaching the problem.
Derill03
Messages
62
Reaction score
0
Matrix A:

1 2 4 1
2 4 8 2
3 1 5 7

The question says find a basis for the solution set AX=0, X is the vector of variables
[x1,x2,x3,x4]^t

What is a basis? and how can i approach this problem?
 
Physics news on Phys.org
By inspection, I can see that the solution space for the equation AX = 0 will be at least a one-dimensional subspace of R4 (i.e., a line through the origin), and on closer inspection I can see that this solution space will be a two-dimensional subspace of R4, a plane through the origin.

You asked what a basis is. Isn't that term defined in your textbook? What this problem is asking for is a set of vectors that spans the solution space. IOW, a set of vectors such that any solution vector is a linear combination of the basis vectors.

You should also look up the definitions of the terms I have underlined.
 
Solve the equations x+ 2y+ 4z+ u= 0, 2x+ 4y+ 8z+ u= 0, and 3x+ y+ 5z+ 7u= 0. There is, of course, an infinite number of solutions so instead of a single solution you will get equations expressing some of the variables in terms of the others. Choose simple values for those "others" and solve for the rest.
 
Question: A clock's minute hand has length 4 and its hour hand has length 3. What is the distance between the tips at the moment when it is increasing most rapidly?(Putnam Exam Question) Answer: Making assumption that both the hands moves at constant angular velocities, the answer is ## \sqrt{7} .## But don't you think this assumption is somewhat doubtful and wrong?

Similar threads

  • · Replies 9 ·
Replies
9
Views
3K
  • · Replies 14 ·
Replies
14
Views
3K
  • · Replies 1 ·
Replies
1
Views
2K
Replies
4
Views
2K
  • · Replies 58 ·
2
Replies
58
Views
4K
  • · Replies 24 ·
Replies
24
Views
3K
  • · Replies 5 ·
Replies
5
Views
3K
  • · Replies 2 ·
Replies
2
Views
2K
Replies
2
Views
2K
  • · Replies 1 ·
Replies
1
Views
1K